Computing the matrix powers of a non-diagonalizable matrixLimit of powers of $3times3$ matrixDiagonalizable A, computing fast.How to prove that A is diagonalizable?Is this matrix diagonalizable over $mathbbR$ or $mathbbC$?Computing matrix exponential of non-diagonalizable 2x2 matrixMatrix functions of a non-diagonalizable matrixFinding the Exponential of a Matrix that is not DiagonalizableIs following matrix diagonalizable?A demonstration on diagonalizable matrixFor which numbers is the matrix diagonalizable?Show that for all $mge 2$ there is some invertible, non-diagonalizable matrix $Ain M_2(mathbbR)$ s.t $A^m$ is diagonalizable.

When and what was the first 3D acceleration device ever released?

What to do when you've set the wrong ISO for your film?

How strong are Wi-Fi signals?

Plot twist where the antagonist wins

Why aren't space telescopes put in GEO?

Is it rude to call a professor by their last name with no prefix in a non-academic setting?

Why does the 6502 have the BIT instruction?

At what point in European history could a government build a printing press given a basic description?

Is neural networks training done one-by-one?

Is it possible to play as a necromancer skeleton?

Ticket to ride, 1910: What are the big cities

What is the largest (size) solid object ever dropped from an airplane to impact the ground in freefall?

Why are C64 games inconsistent with which joystick port they use?

What are the real benefits of using Salesforce DX?

Why do airplanes use an axial flow jet engine instead of a more compact centrifugal jet engine?

Installed Electric Tankless Water Heater - Internet loss when active

Computing the matrix powers of a non-diagonalizable matrix

Is it true that cut time means "play twice as fast as written"?

Who will lead the country until there is a new Tory leader?

Where is the logic in castrating fighters?

the meaning of 'carry' in a novel

Have 1.5% of all nuclear reactors ever built melted down?

Should breaking down something like a door be adjudicated as an attempt to beat its AC and HP, or as an ability check against a set DC?

If a person had control of every single cell of their body, would they be able to transform into another creature?



Computing the matrix powers of a non-diagonalizable matrix


Limit of powers of $3times3$ matrixDiagonalizable A, computing fast.How to prove that A is diagonalizable?Is this matrix diagonalizable over $mathbbR$ or $mathbbC$?Computing matrix exponential of non-diagonalizable 2x2 matrixMatrix functions of a non-diagonalizable matrixFinding the Exponential of a Matrix that is not DiagonalizableIs following matrix diagonalizable?A demonstration on diagonalizable matrixFor which numbers is the matrix diagonalizable?Show that for all $mge 2$ there is some invertible, non-diagonalizable matrix $Ain M_2(mathbbR)$ s.t $A^m$ is diagonalizable.













4












$begingroup$


Define
beginequation
A = beginpmatrix frac12 &frac12 & 0\ 0& frac34 & frac14\ 0& frac14 & frac34 endpmatrix.
endequation



Note that the sum of the dimensions of the eigenspaces of $A$ is only two. $A$ is thus not diagonalizable. How can we compute $A^n$?










share|cite|improve this question









$endgroup$







  • 1




    $begingroup$
    Use Cayley-Hamilton to write $A^n=aI+bA+cA^2$ and then use the eigenvalues to find the coefficients.
    $endgroup$
    – amd
    8 hours ago






  • 1




    $begingroup$
    Why is this question getting upvotes? It show no effort to solve the problem and aside from a trivial difference in the specific values of the matrix elements is a duplicate of many other previous questions.
    $endgroup$
    – amd
    8 hours ago






  • 3




    $begingroup$
    @amd Because people can upvote or downvote freely.
    $endgroup$
    – DonAntonio
    7 hours ago






  • 2




    $begingroup$
    @amd it would be great if you linked some of these questions that seem to already have good answers to my question. Also, I don’t show effort to solve the problem because I answered my own question below.
    $endgroup$
    – Maximilian Janisch
    7 hours ago











  • $begingroup$
    Yes, I hadn’t noticed that you had answered your own question. I withdraw that part of my comment.
    $endgroup$
    – amd
    3 hours ago















4












$begingroup$


Define
beginequation
A = beginpmatrix frac12 &frac12 & 0\ 0& frac34 & frac14\ 0& frac14 & frac34 endpmatrix.
endequation



Note that the sum of the dimensions of the eigenspaces of $A$ is only two. $A$ is thus not diagonalizable. How can we compute $A^n$?










share|cite|improve this question









$endgroup$







  • 1




    $begingroup$
    Use Cayley-Hamilton to write $A^n=aI+bA+cA^2$ and then use the eigenvalues to find the coefficients.
    $endgroup$
    – amd
    8 hours ago






  • 1




    $begingroup$
    Why is this question getting upvotes? It show no effort to solve the problem and aside from a trivial difference in the specific values of the matrix elements is a duplicate of many other previous questions.
    $endgroup$
    – amd
    8 hours ago






  • 3




    $begingroup$
    @amd Because people can upvote or downvote freely.
    $endgroup$
    – DonAntonio
    7 hours ago






  • 2




    $begingroup$
    @amd it would be great if you linked some of these questions that seem to already have good answers to my question. Also, I don’t show effort to solve the problem because I answered my own question below.
    $endgroup$
    – Maximilian Janisch
    7 hours ago











  • $begingroup$
    Yes, I hadn’t noticed that you had answered your own question. I withdraw that part of my comment.
    $endgroup$
    – amd
    3 hours ago













4












4








4





$begingroup$


Define
beginequation
A = beginpmatrix frac12 &frac12 & 0\ 0& frac34 & frac14\ 0& frac14 & frac34 endpmatrix.
endequation



Note that the sum of the dimensions of the eigenspaces of $A$ is only two. $A$ is thus not diagonalizable. How can we compute $A^n$?










share|cite|improve this question









$endgroup$




Define
beginequation
A = beginpmatrix frac12 &frac12 & 0\ 0& frac34 & frac14\ 0& frac14 & frac34 endpmatrix.
endequation



Note that the sum of the dimensions of the eigenspaces of $A$ is only two. $A$ is thus not diagonalizable. How can we compute $A^n$?







linear-algebra matrices matrix-exponential






share|cite|improve this question













share|cite|improve this question











share|cite|improve this question




share|cite|improve this question










asked 8 hours ago









Maximilian JanischMaximilian Janisch

1,52718




1,52718







  • 1




    $begingroup$
    Use Cayley-Hamilton to write $A^n=aI+bA+cA^2$ and then use the eigenvalues to find the coefficients.
    $endgroup$
    – amd
    8 hours ago






  • 1




    $begingroup$
    Why is this question getting upvotes? It show no effort to solve the problem and aside from a trivial difference in the specific values of the matrix elements is a duplicate of many other previous questions.
    $endgroup$
    – amd
    8 hours ago






  • 3




    $begingroup$
    @amd Because people can upvote or downvote freely.
    $endgroup$
    – DonAntonio
    7 hours ago






  • 2




    $begingroup$
    @amd it would be great if you linked some of these questions that seem to already have good answers to my question. Also, I don’t show effort to solve the problem because I answered my own question below.
    $endgroup$
    – Maximilian Janisch
    7 hours ago











  • $begingroup$
    Yes, I hadn’t noticed that you had answered your own question. I withdraw that part of my comment.
    $endgroup$
    – amd
    3 hours ago












  • 1




    $begingroup$
    Use Cayley-Hamilton to write $A^n=aI+bA+cA^2$ and then use the eigenvalues to find the coefficients.
    $endgroup$
    – amd
    8 hours ago






  • 1




    $begingroup$
    Why is this question getting upvotes? It show no effort to solve the problem and aside from a trivial difference in the specific values of the matrix elements is a duplicate of many other previous questions.
    $endgroup$
    – amd
    8 hours ago






  • 3




    $begingroup$
    @amd Because people can upvote or downvote freely.
    $endgroup$
    – DonAntonio
    7 hours ago






  • 2




    $begingroup$
    @amd it would be great if you linked some of these questions that seem to already have good answers to my question. Also, I don’t show effort to solve the problem because I answered my own question below.
    $endgroup$
    – Maximilian Janisch
    7 hours ago











  • $begingroup$
    Yes, I hadn’t noticed that you had answered your own question. I withdraw that part of my comment.
    $endgroup$
    – amd
    3 hours ago







1




1




$begingroup$
Use Cayley-Hamilton to write $A^n=aI+bA+cA^2$ and then use the eigenvalues to find the coefficients.
$endgroup$
– amd
8 hours ago




$begingroup$
Use Cayley-Hamilton to write $A^n=aI+bA+cA^2$ and then use the eigenvalues to find the coefficients.
$endgroup$
– amd
8 hours ago




1




1




$begingroup$
Why is this question getting upvotes? It show no effort to solve the problem and aside from a trivial difference in the specific values of the matrix elements is a duplicate of many other previous questions.
$endgroup$
– amd
8 hours ago




$begingroup$
Why is this question getting upvotes? It show no effort to solve the problem and aside from a trivial difference in the specific values of the matrix elements is a duplicate of many other previous questions.
$endgroup$
– amd
8 hours ago




3




3




$begingroup$
@amd Because people can upvote or downvote freely.
$endgroup$
– DonAntonio
7 hours ago




$begingroup$
@amd Because people can upvote or downvote freely.
$endgroup$
– DonAntonio
7 hours ago




2




2




$begingroup$
@amd it would be great if you linked some of these questions that seem to already have good answers to my question. Also, I don’t show effort to solve the problem because I answered my own question below.
$endgroup$
– Maximilian Janisch
7 hours ago





$begingroup$
@amd it would be great if you linked some of these questions that seem to already have good answers to my question. Also, I don’t show effort to solve the problem because I answered my own question below.
$endgroup$
– Maximilian Janisch
7 hours ago













$begingroup$
Yes, I hadn’t noticed that you had answered your own question. I withdraw that part of my comment.
$endgroup$
– amd
3 hours ago




$begingroup$
Yes, I hadn’t noticed that you had answered your own question. I withdraw that part of my comment.
$endgroup$
– amd
3 hours ago










2 Answers
2






active

oldest

votes


















4












$begingroup$

Here is a different way using a rather classical trick, converting the issue into a binomial expansion. Indeed, we can write :



$$A=frac12(I+B) textwhere B:=beginpmatrix0&1&0 \0&1/2&1/2\0&1/2&1/2endpmatrix$$



where matrix $B$ has the following particularity



$$B^n=C textfor all n>1 textwhere C:=beginpmatrix0&1/2&1/2\0&1/2&1/2\0&1/2&1/2endpmatrix$$



Therefore



$$A^n = dfrac12^nleft(I+binomn1B+binomn2B^2+binomn3B^3+cdots+binomnnB^nright)$$



$$A^n = dfrac12^nleft(I+nB+binomn2C+binomn3C+cdots+binomnnCright)tag1$$



As is well known, $sum_k=0^n binomnk=2^n$, reducing (1) to :



$$A^n = dfrac12^nleft(I+nB+(2^n-n-1)Cright)$$



It suffices now to replace $B$ and $C$ by their expression



$$A^n = dfrac12^nleft(beginpmatrix1&0&0\0&1&0\0&0&1endpmatrix+nbeginpmatrix0&1&0 \0&1/2&1/2\0&1/2&1/2endpmatrix+(2^n-n-1)beginpmatrix0&1/2&1/2\0&1/2&1/2\0&1/2&1/2endpmatrixright)$$



to get the result (coinciding with yours !).






share|cite|improve this answer









$endgroup$








  • 1




    $begingroup$
    That is very nice indeed!
    $endgroup$
    – Maximilian Janisch
    3 hours ago






  • 3




    $begingroup$
    I have been able to find such a decomposition guided by the fact that $A$ is a stochastic matrix.
    $endgroup$
    – Jean Marie
    3 hours ago










  • $begingroup$
    Thanks for your appreciation.
    $endgroup$
    – Jean Marie
    2 hours ago










  • $begingroup$
    I definitely learned something from your answer (I accidentally deleted my previous comment)
    $endgroup$
    – Maximilian Janisch
    2 hours ago






  • 2




    $begingroup$
    Something I haven't said is that $(A+B)^n$ can be expanded using the binomial theorem whenever $A$ and $B$ commute. This is the case in this problem, with $A=I$ which commutes with any matrix.
    $endgroup$
    – Jean Marie
    2 hours ago


















5












$begingroup$

Note that your matrix $A$ has the generalized eigenvectors



beginequationv_1=beginpmatrix1 \ 0 \ 0endpmatrix, v_2=beginpmatrix0 \ 2 \ -2endpmatrix, v_3=beginpmatrix0\ 0 \ 1endpmatrix.endequation



Thus, by Jordan decomposition, $A=big(v_1,v_2,v_3big)Jbig(v_1,v_2,v_3big)^-1$, where



beginequationJ=beginpmatrixfrac12 & 1 & 0\0 & frac12 & 0\0 & 0 & 1endpmatrix.endequation



The problem of calculating $A^n$ is thus reduced to calculating $J^n$. Let $a_ij^(n)$ denote the entry of $J^n$ in the $i$-th row and $j$-th column.



The product of an arbitrary $3times3$-matrix with $J$ is given by:
beginequation
beginpmatrix
a&b&c\d&e&f\g&h&i
endpmatrix J =
beginpmatrix
frac a2&a+frac b2&c\frac d2&d+frac e2&f\frac g2&g+frac h2&i
endpmatrix.
endequation



We can deduce that, for all $ninBbb N$:
beginalign
a_11^(n)&=a_22^(n)=frac12^n, \a_21^(n)&=a_31^(n)=0,\
a_13^(n)&=a_23^(n)=a_32^(n)=0, \
a_33^(n)&=1,\
a_12^(n+1)&=a_11^(n)+fraca_12^(n)2=frac12^n+fraca_12^(n)2.
endalign



Thus, all $a_ij^(n)$ are explicitly known except for $a_12^(n)$. Note that, by the last equation, beginequationa_12^(n+1)=2^-n+fraca_12^(n)2 = 2^-n+2^-n+fraca_12^(n-1)4 = dots = (n+1)cdot2^-n.endequation



Thus,
beginequationJ^n=beginpmatrix2^-n&ncdot 2^1-n & 0\0 & 2^-n & 0\0 & 0 & 1endpmatrix.endequation



And by some calculations, we find that
beginequation
A^n=big(v_1,v_2,v_3big)J^nbig(v_1,v_2,v_3big)^-1=
beginpmatrix
2^-n & ncdot 2^-n-1 - 2^-n-1 + frac12 & 1-fracn+12^nover2\
0 & 2^-n+1over2 & 1-2^-nover2 \
0 & 1-2^-nover2 & 2^-n+1over2
endpmatrix.
endequation






share|cite|improve this answer









$endgroup$












  • $begingroup$
    I don't understand very well the "your" in the first sentence of this answer "Note that your matrix..." ; in fact, as far I have understood, you are answering your own question ...
    $endgroup$
    – Jean Marie
    7 hours ago







  • 1




    $begingroup$
    @JeanMarie indeed I am answering my own question. On the questions on this forum that I saw that were answered by the OP, the OP was always talking about himself in second person; so I decided to do the same. Feel free to edit this if that is actually a wrong decision on my part. PS: Another factor in this is that I first wanted this to be a part of my answer to this question, but I decided to outsource it
    $endgroup$
    – Maximilian Janisch
    7 hours ago











  • $begingroup$
    Thanks for your answer... I think you have understood it as humor.
    $endgroup$
    – Jean Marie
    7 hours ago










  • $begingroup$
    I think that you can simplify this quite a bit by noting that $J=D+N$, where $D$ is diagonal and $N$ is nilpotent of order 2. $D$ and $N$ commute, so expand using the Binomial Theorem: $(D+N)^n=D^n+nND^n-1$. Powers of $D$ are themselves diagonal, so the second term should be quite simple to compute.
    $endgroup$
    – amd
    1 hour ago












Your Answer








StackExchange.ready(function()
var channelOptions =
tags: "".split(" "),
id: "69"
;
initTagRenderer("".split(" "), "".split(" "), channelOptions);

StackExchange.using("externalEditor", function()
// Have to fire editor after snippets, if snippets enabled
if (StackExchange.settings.snippets.snippetsEnabled)
StackExchange.using("snippets", function()
createEditor();
);

else
createEditor();

);

function createEditor()
StackExchange.prepareEditor(
heartbeatType: 'answer',
autoActivateHeartbeat: false,
convertImagesToLinks: true,
noModals: true,
showLowRepImageUploadWarning: true,
reputationToPostImages: 10,
bindNavPrevention: true,
postfix: "",
imageUploader:
brandingHtml: "Powered by u003ca class="icon-imgur-white" href="https://imgur.com/"u003eu003c/au003e",
contentPolicyHtml: "User contributions licensed under u003ca href="https://creativecommons.org/licenses/by-sa/3.0/"u003ecc by-sa 3.0 with attribution requiredu003c/au003e u003ca href="https://stackoverflow.com/legal/content-policy"u003e(content policy)u003c/au003e",
allowUrls: true
,
noCode: true, onDemand: true,
discardSelector: ".discard-answer"
,immediatelyShowMarkdownHelp:true
);



);













draft saved

draft discarded


















StackExchange.ready(
function ()
StackExchange.openid.initPostLogin('.new-post-login', 'https%3a%2f%2fmath.stackexchange.com%2fquestions%2f3239468%2fcomputing-the-matrix-powers-of-a-non-diagonalizable-matrix%23new-answer', 'question_page');

);

Post as a guest















Required, but never shown

























2 Answers
2






active

oldest

votes








2 Answers
2






active

oldest

votes









active

oldest

votes






active

oldest

votes









4












$begingroup$

Here is a different way using a rather classical trick, converting the issue into a binomial expansion. Indeed, we can write :



$$A=frac12(I+B) textwhere B:=beginpmatrix0&1&0 \0&1/2&1/2\0&1/2&1/2endpmatrix$$



where matrix $B$ has the following particularity



$$B^n=C textfor all n>1 textwhere C:=beginpmatrix0&1/2&1/2\0&1/2&1/2\0&1/2&1/2endpmatrix$$



Therefore



$$A^n = dfrac12^nleft(I+binomn1B+binomn2B^2+binomn3B^3+cdots+binomnnB^nright)$$



$$A^n = dfrac12^nleft(I+nB+binomn2C+binomn3C+cdots+binomnnCright)tag1$$



As is well known, $sum_k=0^n binomnk=2^n$, reducing (1) to :



$$A^n = dfrac12^nleft(I+nB+(2^n-n-1)Cright)$$



It suffices now to replace $B$ and $C$ by their expression



$$A^n = dfrac12^nleft(beginpmatrix1&0&0\0&1&0\0&0&1endpmatrix+nbeginpmatrix0&1&0 \0&1/2&1/2\0&1/2&1/2endpmatrix+(2^n-n-1)beginpmatrix0&1/2&1/2\0&1/2&1/2\0&1/2&1/2endpmatrixright)$$



to get the result (coinciding with yours !).






share|cite|improve this answer









$endgroup$








  • 1




    $begingroup$
    That is very nice indeed!
    $endgroup$
    – Maximilian Janisch
    3 hours ago






  • 3




    $begingroup$
    I have been able to find such a decomposition guided by the fact that $A$ is a stochastic matrix.
    $endgroup$
    – Jean Marie
    3 hours ago










  • $begingroup$
    Thanks for your appreciation.
    $endgroup$
    – Jean Marie
    2 hours ago










  • $begingroup$
    I definitely learned something from your answer (I accidentally deleted my previous comment)
    $endgroup$
    – Maximilian Janisch
    2 hours ago






  • 2




    $begingroup$
    Something I haven't said is that $(A+B)^n$ can be expanded using the binomial theorem whenever $A$ and $B$ commute. This is the case in this problem, with $A=I$ which commutes with any matrix.
    $endgroup$
    – Jean Marie
    2 hours ago















4












$begingroup$

Here is a different way using a rather classical trick, converting the issue into a binomial expansion. Indeed, we can write :



$$A=frac12(I+B) textwhere B:=beginpmatrix0&1&0 \0&1/2&1/2\0&1/2&1/2endpmatrix$$



where matrix $B$ has the following particularity



$$B^n=C textfor all n>1 textwhere C:=beginpmatrix0&1/2&1/2\0&1/2&1/2\0&1/2&1/2endpmatrix$$



Therefore



$$A^n = dfrac12^nleft(I+binomn1B+binomn2B^2+binomn3B^3+cdots+binomnnB^nright)$$



$$A^n = dfrac12^nleft(I+nB+binomn2C+binomn3C+cdots+binomnnCright)tag1$$



As is well known, $sum_k=0^n binomnk=2^n$, reducing (1) to :



$$A^n = dfrac12^nleft(I+nB+(2^n-n-1)Cright)$$



It suffices now to replace $B$ and $C$ by their expression



$$A^n = dfrac12^nleft(beginpmatrix1&0&0\0&1&0\0&0&1endpmatrix+nbeginpmatrix0&1&0 \0&1/2&1/2\0&1/2&1/2endpmatrix+(2^n-n-1)beginpmatrix0&1/2&1/2\0&1/2&1/2\0&1/2&1/2endpmatrixright)$$



to get the result (coinciding with yours !).






share|cite|improve this answer









$endgroup$








  • 1




    $begingroup$
    That is very nice indeed!
    $endgroup$
    – Maximilian Janisch
    3 hours ago






  • 3




    $begingroup$
    I have been able to find such a decomposition guided by the fact that $A$ is a stochastic matrix.
    $endgroup$
    – Jean Marie
    3 hours ago










  • $begingroup$
    Thanks for your appreciation.
    $endgroup$
    – Jean Marie
    2 hours ago










  • $begingroup$
    I definitely learned something from your answer (I accidentally deleted my previous comment)
    $endgroup$
    – Maximilian Janisch
    2 hours ago






  • 2




    $begingroup$
    Something I haven't said is that $(A+B)^n$ can be expanded using the binomial theorem whenever $A$ and $B$ commute. This is the case in this problem, with $A=I$ which commutes with any matrix.
    $endgroup$
    – Jean Marie
    2 hours ago













4












4








4





$begingroup$

Here is a different way using a rather classical trick, converting the issue into a binomial expansion. Indeed, we can write :



$$A=frac12(I+B) textwhere B:=beginpmatrix0&1&0 \0&1/2&1/2\0&1/2&1/2endpmatrix$$



where matrix $B$ has the following particularity



$$B^n=C textfor all n>1 textwhere C:=beginpmatrix0&1/2&1/2\0&1/2&1/2\0&1/2&1/2endpmatrix$$



Therefore



$$A^n = dfrac12^nleft(I+binomn1B+binomn2B^2+binomn3B^3+cdots+binomnnB^nright)$$



$$A^n = dfrac12^nleft(I+nB+binomn2C+binomn3C+cdots+binomnnCright)tag1$$



As is well known, $sum_k=0^n binomnk=2^n$, reducing (1) to :



$$A^n = dfrac12^nleft(I+nB+(2^n-n-1)Cright)$$



It suffices now to replace $B$ and $C$ by their expression



$$A^n = dfrac12^nleft(beginpmatrix1&0&0\0&1&0\0&0&1endpmatrix+nbeginpmatrix0&1&0 \0&1/2&1/2\0&1/2&1/2endpmatrix+(2^n-n-1)beginpmatrix0&1/2&1/2\0&1/2&1/2\0&1/2&1/2endpmatrixright)$$



to get the result (coinciding with yours !).






share|cite|improve this answer









$endgroup$



Here is a different way using a rather classical trick, converting the issue into a binomial expansion. Indeed, we can write :



$$A=frac12(I+B) textwhere B:=beginpmatrix0&1&0 \0&1/2&1/2\0&1/2&1/2endpmatrix$$



where matrix $B$ has the following particularity



$$B^n=C textfor all n>1 textwhere C:=beginpmatrix0&1/2&1/2\0&1/2&1/2\0&1/2&1/2endpmatrix$$



Therefore



$$A^n = dfrac12^nleft(I+binomn1B+binomn2B^2+binomn3B^3+cdots+binomnnB^nright)$$



$$A^n = dfrac12^nleft(I+nB+binomn2C+binomn3C+cdots+binomnnCright)tag1$$



As is well known, $sum_k=0^n binomnk=2^n$, reducing (1) to :



$$A^n = dfrac12^nleft(I+nB+(2^n-n-1)Cright)$$



It suffices now to replace $B$ and $C$ by their expression



$$A^n = dfrac12^nleft(beginpmatrix1&0&0\0&1&0\0&0&1endpmatrix+nbeginpmatrix0&1&0 \0&1/2&1/2\0&1/2&1/2endpmatrix+(2^n-n-1)beginpmatrix0&1/2&1/2\0&1/2&1/2\0&1/2&1/2endpmatrixright)$$



to get the result (coinciding with yours !).







share|cite|improve this answer












share|cite|improve this answer



share|cite|improve this answer










answered 3 hours ago









Jean MarieJean Marie

33k42357




33k42357







  • 1




    $begingroup$
    That is very nice indeed!
    $endgroup$
    – Maximilian Janisch
    3 hours ago






  • 3




    $begingroup$
    I have been able to find such a decomposition guided by the fact that $A$ is a stochastic matrix.
    $endgroup$
    – Jean Marie
    3 hours ago










  • $begingroup$
    Thanks for your appreciation.
    $endgroup$
    – Jean Marie
    2 hours ago










  • $begingroup$
    I definitely learned something from your answer (I accidentally deleted my previous comment)
    $endgroup$
    – Maximilian Janisch
    2 hours ago






  • 2




    $begingroup$
    Something I haven't said is that $(A+B)^n$ can be expanded using the binomial theorem whenever $A$ and $B$ commute. This is the case in this problem, with $A=I$ which commutes with any matrix.
    $endgroup$
    – Jean Marie
    2 hours ago












  • 1




    $begingroup$
    That is very nice indeed!
    $endgroup$
    – Maximilian Janisch
    3 hours ago






  • 3




    $begingroup$
    I have been able to find such a decomposition guided by the fact that $A$ is a stochastic matrix.
    $endgroup$
    – Jean Marie
    3 hours ago










  • $begingroup$
    Thanks for your appreciation.
    $endgroup$
    – Jean Marie
    2 hours ago










  • $begingroup$
    I definitely learned something from your answer (I accidentally deleted my previous comment)
    $endgroup$
    – Maximilian Janisch
    2 hours ago






  • 2




    $begingroup$
    Something I haven't said is that $(A+B)^n$ can be expanded using the binomial theorem whenever $A$ and $B$ commute. This is the case in this problem, with $A=I$ which commutes with any matrix.
    $endgroup$
    – Jean Marie
    2 hours ago







1




1




$begingroup$
That is very nice indeed!
$endgroup$
– Maximilian Janisch
3 hours ago




$begingroup$
That is very nice indeed!
$endgroup$
– Maximilian Janisch
3 hours ago




3




3




$begingroup$
I have been able to find such a decomposition guided by the fact that $A$ is a stochastic matrix.
$endgroup$
– Jean Marie
3 hours ago




$begingroup$
I have been able to find such a decomposition guided by the fact that $A$ is a stochastic matrix.
$endgroup$
– Jean Marie
3 hours ago












$begingroup$
Thanks for your appreciation.
$endgroup$
– Jean Marie
2 hours ago




$begingroup$
Thanks for your appreciation.
$endgroup$
– Jean Marie
2 hours ago












$begingroup$
I definitely learned something from your answer (I accidentally deleted my previous comment)
$endgroup$
– Maximilian Janisch
2 hours ago




$begingroup$
I definitely learned something from your answer (I accidentally deleted my previous comment)
$endgroup$
– Maximilian Janisch
2 hours ago




2




2




$begingroup$
Something I haven't said is that $(A+B)^n$ can be expanded using the binomial theorem whenever $A$ and $B$ commute. This is the case in this problem, with $A=I$ which commutes with any matrix.
$endgroup$
– Jean Marie
2 hours ago




$begingroup$
Something I haven't said is that $(A+B)^n$ can be expanded using the binomial theorem whenever $A$ and $B$ commute. This is the case in this problem, with $A=I$ which commutes with any matrix.
$endgroup$
– Jean Marie
2 hours ago











5












$begingroup$

Note that your matrix $A$ has the generalized eigenvectors



beginequationv_1=beginpmatrix1 \ 0 \ 0endpmatrix, v_2=beginpmatrix0 \ 2 \ -2endpmatrix, v_3=beginpmatrix0\ 0 \ 1endpmatrix.endequation



Thus, by Jordan decomposition, $A=big(v_1,v_2,v_3big)Jbig(v_1,v_2,v_3big)^-1$, where



beginequationJ=beginpmatrixfrac12 & 1 & 0\0 & frac12 & 0\0 & 0 & 1endpmatrix.endequation



The problem of calculating $A^n$ is thus reduced to calculating $J^n$. Let $a_ij^(n)$ denote the entry of $J^n$ in the $i$-th row and $j$-th column.



The product of an arbitrary $3times3$-matrix with $J$ is given by:
beginequation
beginpmatrix
a&b&c\d&e&f\g&h&i
endpmatrix J =
beginpmatrix
frac a2&a+frac b2&c\frac d2&d+frac e2&f\frac g2&g+frac h2&i
endpmatrix.
endequation



We can deduce that, for all $ninBbb N$:
beginalign
a_11^(n)&=a_22^(n)=frac12^n, \a_21^(n)&=a_31^(n)=0,\
a_13^(n)&=a_23^(n)=a_32^(n)=0, \
a_33^(n)&=1,\
a_12^(n+1)&=a_11^(n)+fraca_12^(n)2=frac12^n+fraca_12^(n)2.
endalign



Thus, all $a_ij^(n)$ are explicitly known except for $a_12^(n)$. Note that, by the last equation, beginequationa_12^(n+1)=2^-n+fraca_12^(n)2 = 2^-n+2^-n+fraca_12^(n-1)4 = dots = (n+1)cdot2^-n.endequation



Thus,
beginequationJ^n=beginpmatrix2^-n&ncdot 2^1-n & 0\0 & 2^-n & 0\0 & 0 & 1endpmatrix.endequation



And by some calculations, we find that
beginequation
A^n=big(v_1,v_2,v_3big)J^nbig(v_1,v_2,v_3big)^-1=
beginpmatrix
2^-n & ncdot 2^-n-1 - 2^-n-1 + frac12 & 1-fracn+12^nover2\
0 & 2^-n+1over2 & 1-2^-nover2 \
0 & 1-2^-nover2 & 2^-n+1over2
endpmatrix.
endequation






share|cite|improve this answer









$endgroup$












  • $begingroup$
    I don't understand very well the "your" in the first sentence of this answer "Note that your matrix..." ; in fact, as far I have understood, you are answering your own question ...
    $endgroup$
    – Jean Marie
    7 hours ago







  • 1




    $begingroup$
    @JeanMarie indeed I am answering my own question. On the questions on this forum that I saw that were answered by the OP, the OP was always talking about himself in second person; so I decided to do the same. Feel free to edit this if that is actually a wrong decision on my part. PS: Another factor in this is that I first wanted this to be a part of my answer to this question, but I decided to outsource it
    $endgroup$
    – Maximilian Janisch
    7 hours ago











  • $begingroup$
    Thanks for your answer... I think you have understood it as humor.
    $endgroup$
    – Jean Marie
    7 hours ago










  • $begingroup$
    I think that you can simplify this quite a bit by noting that $J=D+N$, where $D$ is diagonal and $N$ is nilpotent of order 2. $D$ and $N$ commute, so expand using the Binomial Theorem: $(D+N)^n=D^n+nND^n-1$. Powers of $D$ are themselves diagonal, so the second term should be quite simple to compute.
    $endgroup$
    – amd
    1 hour ago
















5












$begingroup$

Note that your matrix $A$ has the generalized eigenvectors



beginequationv_1=beginpmatrix1 \ 0 \ 0endpmatrix, v_2=beginpmatrix0 \ 2 \ -2endpmatrix, v_3=beginpmatrix0\ 0 \ 1endpmatrix.endequation



Thus, by Jordan decomposition, $A=big(v_1,v_2,v_3big)Jbig(v_1,v_2,v_3big)^-1$, where



beginequationJ=beginpmatrixfrac12 & 1 & 0\0 & frac12 & 0\0 & 0 & 1endpmatrix.endequation



The problem of calculating $A^n$ is thus reduced to calculating $J^n$. Let $a_ij^(n)$ denote the entry of $J^n$ in the $i$-th row and $j$-th column.



The product of an arbitrary $3times3$-matrix with $J$ is given by:
beginequation
beginpmatrix
a&b&c\d&e&f\g&h&i
endpmatrix J =
beginpmatrix
frac a2&a+frac b2&c\frac d2&d+frac e2&f\frac g2&g+frac h2&i
endpmatrix.
endequation



We can deduce that, for all $ninBbb N$:
beginalign
a_11^(n)&=a_22^(n)=frac12^n, \a_21^(n)&=a_31^(n)=0,\
a_13^(n)&=a_23^(n)=a_32^(n)=0, \
a_33^(n)&=1,\
a_12^(n+1)&=a_11^(n)+fraca_12^(n)2=frac12^n+fraca_12^(n)2.
endalign



Thus, all $a_ij^(n)$ are explicitly known except for $a_12^(n)$. Note that, by the last equation, beginequationa_12^(n+1)=2^-n+fraca_12^(n)2 = 2^-n+2^-n+fraca_12^(n-1)4 = dots = (n+1)cdot2^-n.endequation



Thus,
beginequationJ^n=beginpmatrix2^-n&ncdot 2^1-n & 0\0 & 2^-n & 0\0 & 0 & 1endpmatrix.endequation



And by some calculations, we find that
beginequation
A^n=big(v_1,v_2,v_3big)J^nbig(v_1,v_2,v_3big)^-1=
beginpmatrix
2^-n & ncdot 2^-n-1 - 2^-n-1 + frac12 & 1-fracn+12^nover2\
0 & 2^-n+1over2 & 1-2^-nover2 \
0 & 1-2^-nover2 & 2^-n+1over2
endpmatrix.
endequation






share|cite|improve this answer









$endgroup$












  • $begingroup$
    I don't understand very well the "your" in the first sentence of this answer "Note that your matrix..." ; in fact, as far I have understood, you are answering your own question ...
    $endgroup$
    – Jean Marie
    7 hours ago







  • 1




    $begingroup$
    @JeanMarie indeed I am answering my own question. On the questions on this forum that I saw that were answered by the OP, the OP was always talking about himself in second person; so I decided to do the same. Feel free to edit this if that is actually a wrong decision on my part. PS: Another factor in this is that I first wanted this to be a part of my answer to this question, but I decided to outsource it
    $endgroup$
    – Maximilian Janisch
    7 hours ago











  • $begingroup$
    Thanks for your answer... I think you have understood it as humor.
    $endgroup$
    – Jean Marie
    7 hours ago










  • $begingroup$
    I think that you can simplify this quite a bit by noting that $J=D+N$, where $D$ is diagonal and $N$ is nilpotent of order 2. $D$ and $N$ commute, so expand using the Binomial Theorem: $(D+N)^n=D^n+nND^n-1$. Powers of $D$ are themselves diagonal, so the second term should be quite simple to compute.
    $endgroup$
    – amd
    1 hour ago














5












5








5





$begingroup$

Note that your matrix $A$ has the generalized eigenvectors



beginequationv_1=beginpmatrix1 \ 0 \ 0endpmatrix, v_2=beginpmatrix0 \ 2 \ -2endpmatrix, v_3=beginpmatrix0\ 0 \ 1endpmatrix.endequation



Thus, by Jordan decomposition, $A=big(v_1,v_2,v_3big)Jbig(v_1,v_2,v_3big)^-1$, where



beginequationJ=beginpmatrixfrac12 & 1 & 0\0 & frac12 & 0\0 & 0 & 1endpmatrix.endequation



The problem of calculating $A^n$ is thus reduced to calculating $J^n$. Let $a_ij^(n)$ denote the entry of $J^n$ in the $i$-th row and $j$-th column.



The product of an arbitrary $3times3$-matrix with $J$ is given by:
beginequation
beginpmatrix
a&b&c\d&e&f\g&h&i
endpmatrix J =
beginpmatrix
frac a2&a+frac b2&c\frac d2&d+frac e2&f\frac g2&g+frac h2&i
endpmatrix.
endequation



We can deduce that, for all $ninBbb N$:
beginalign
a_11^(n)&=a_22^(n)=frac12^n, \a_21^(n)&=a_31^(n)=0,\
a_13^(n)&=a_23^(n)=a_32^(n)=0, \
a_33^(n)&=1,\
a_12^(n+1)&=a_11^(n)+fraca_12^(n)2=frac12^n+fraca_12^(n)2.
endalign



Thus, all $a_ij^(n)$ are explicitly known except for $a_12^(n)$. Note that, by the last equation, beginequationa_12^(n+1)=2^-n+fraca_12^(n)2 = 2^-n+2^-n+fraca_12^(n-1)4 = dots = (n+1)cdot2^-n.endequation



Thus,
beginequationJ^n=beginpmatrix2^-n&ncdot 2^1-n & 0\0 & 2^-n & 0\0 & 0 & 1endpmatrix.endequation



And by some calculations, we find that
beginequation
A^n=big(v_1,v_2,v_3big)J^nbig(v_1,v_2,v_3big)^-1=
beginpmatrix
2^-n & ncdot 2^-n-1 - 2^-n-1 + frac12 & 1-fracn+12^nover2\
0 & 2^-n+1over2 & 1-2^-nover2 \
0 & 1-2^-nover2 & 2^-n+1over2
endpmatrix.
endequation






share|cite|improve this answer









$endgroup$



Note that your matrix $A$ has the generalized eigenvectors



beginequationv_1=beginpmatrix1 \ 0 \ 0endpmatrix, v_2=beginpmatrix0 \ 2 \ -2endpmatrix, v_3=beginpmatrix0\ 0 \ 1endpmatrix.endequation



Thus, by Jordan decomposition, $A=big(v_1,v_2,v_3big)Jbig(v_1,v_2,v_3big)^-1$, where



beginequationJ=beginpmatrixfrac12 & 1 & 0\0 & frac12 & 0\0 & 0 & 1endpmatrix.endequation



The problem of calculating $A^n$ is thus reduced to calculating $J^n$. Let $a_ij^(n)$ denote the entry of $J^n$ in the $i$-th row and $j$-th column.



The product of an arbitrary $3times3$-matrix with $J$ is given by:
beginequation
beginpmatrix
a&b&c\d&e&f\g&h&i
endpmatrix J =
beginpmatrix
frac a2&a+frac b2&c\frac d2&d+frac e2&f\frac g2&g+frac h2&i
endpmatrix.
endequation



We can deduce that, for all $ninBbb N$:
beginalign
a_11^(n)&=a_22^(n)=frac12^n, \a_21^(n)&=a_31^(n)=0,\
a_13^(n)&=a_23^(n)=a_32^(n)=0, \
a_33^(n)&=1,\
a_12^(n+1)&=a_11^(n)+fraca_12^(n)2=frac12^n+fraca_12^(n)2.
endalign



Thus, all $a_ij^(n)$ are explicitly known except for $a_12^(n)$. Note that, by the last equation, beginequationa_12^(n+1)=2^-n+fraca_12^(n)2 = 2^-n+2^-n+fraca_12^(n-1)4 = dots = (n+1)cdot2^-n.endequation



Thus,
beginequationJ^n=beginpmatrix2^-n&ncdot 2^1-n & 0\0 & 2^-n & 0\0 & 0 & 1endpmatrix.endequation



And by some calculations, we find that
beginequation
A^n=big(v_1,v_2,v_3big)J^nbig(v_1,v_2,v_3big)^-1=
beginpmatrix
2^-n & ncdot 2^-n-1 - 2^-n-1 + frac12 & 1-fracn+12^nover2\
0 & 2^-n+1over2 & 1-2^-nover2 \
0 & 1-2^-nover2 & 2^-n+1over2
endpmatrix.
endequation







share|cite|improve this answer












share|cite|improve this answer



share|cite|improve this answer










answered 8 hours ago









Maximilian JanischMaximilian Janisch

1,52718




1,52718











  • $begingroup$
    I don't understand very well the "your" in the first sentence of this answer "Note that your matrix..." ; in fact, as far I have understood, you are answering your own question ...
    $endgroup$
    – Jean Marie
    7 hours ago







  • 1




    $begingroup$
    @JeanMarie indeed I am answering my own question. On the questions on this forum that I saw that were answered by the OP, the OP was always talking about himself in second person; so I decided to do the same. Feel free to edit this if that is actually a wrong decision on my part. PS: Another factor in this is that I first wanted this to be a part of my answer to this question, but I decided to outsource it
    $endgroup$
    – Maximilian Janisch
    7 hours ago











  • $begingroup$
    Thanks for your answer... I think you have understood it as humor.
    $endgroup$
    – Jean Marie
    7 hours ago










  • $begingroup$
    I think that you can simplify this quite a bit by noting that $J=D+N$, where $D$ is diagonal and $N$ is nilpotent of order 2. $D$ and $N$ commute, so expand using the Binomial Theorem: $(D+N)^n=D^n+nND^n-1$. Powers of $D$ are themselves diagonal, so the second term should be quite simple to compute.
    $endgroup$
    – amd
    1 hour ago

















  • $begingroup$
    I don't understand very well the "your" in the first sentence of this answer "Note that your matrix..." ; in fact, as far I have understood, you are answering your own question ...
    $endgroup$
    – Jean Marie
    7 hours ago







  • 1




    $begingroup$
    @JeanMarie indeed I am answering my own question. On the questions on this forum that I saw that were answered by the OP, the OP was always talking about himself in second person; so I decided to do the same. Feel free to edit this if that is actually a wrong decision on my part. PS: Another factor in this is that I first wanted this to be a part of my answer to this question, but I decided to outsource it
    $endgroup$
    – Maximilian Janisch
    7 hours ago











  • $begingroup$
    Thanks for your answer... I think you have understood it as humor.
    $endgroup$
    – Jean Marie
    7 hours ago










  • $begingroup$
    I think that you can simplify this quite a bit by noting that $J=D+N$, where $D$ is diagonal and $N$ is nilpotent of order 2. $D$ and $N$ commute, so expand using the Binomial Theorem: $(D+N)^n=D^n+nND^n-1$. Powers of $D$ are themselves diagonal, so the second term should be quite simple to compute.
    $endgroup$
    – amd
    1 hour ago
















$begingroup$
I don't understand very well the "your" in the first sentence of this answer "Note that your matrix..." ; in fact, as far I have understood, you are answering your own question ...
$endgroup$
– Jean Marie
7 hours ago





$begingroup$
I don't understand very well the "your" in the first sentence of this answer "Note that your matrix..." ; in fact, as far I have understood, you are answering your own question ...
$endgroup$
– Jean Marie
7 hours ago





1




1




$begingroup$
@JeanMarie indeed I am answering my own question. On the questions on this forum that I saw that were answered by the OP, the OP was always talking about himself in second person; so I decided to do the same. Feel free to edit this if that is actually a wrong decision on my part. PS: Another factor in this is that I first wanted this to be a part of my answer to this question, but I decided to outsource it
$endgroup$
– Maximilian Janisch
7 hours ago





$begingroup$
@JeanMarie indeed I am answering my own question. On the questions on this forum that I saw that were answered by the OP, the OP was always talking about himself in second person; so I decided to do the same. Feel free to edit this if that is actually a wrong decision on my part. PS: Another factor in this is that I first wanted this to be a part of my answer to this question, but I decided to outsource it
$endgroup$
– Maximilian Janisch
7 hours ago













$begingroup$
Thanks for your answer... I think you have understood it as humor.
$endgroup$
– Jean Marie
7 hours ago




$begingroup$
Thanks for your answer... I think you have understood it as humor.
$endgroup$
– Jean Marie
7 hours ago












$begingroup$
I think that you can simplify this quite a bit by noting that $J=D+N$, where $D$ is diagonal and $N$ is nilpotent of order 2. $D$ and $N$ commute, so expand using the Binomial Theorem: $(D+N)^n=D^n+nND^n-1$. Powers of $D$ are themselves diagonal, so the second term should be quite simple to compute.
$endgroup$
– amd
1 hour ago





$begingroup$
I think that you can simplify this quite a bit by noting that $J=D+N$, where $D$ is diagonal and $N$ is nilpotent of order 2. $D$ and $N$ commute, so expand using the Binomial Theorem: $(D+N)^n=D^n+nND^n-1$. Powers of $D$ are themselves diagonal, so the second term should be quite simple to compute.
$endgroup$
– amd
1 hour ago


















draft saved

draft discarded
















































Thanks for contributing an answer to Mathematics Stack Exchange!


  • Please be sure to answer the question. Provide details and share your research!

But avoid


  • Asking for help, clarification, or responding to other answers.

  • Making statements based on opinion; back them up with references or personal experience.

Use MathJax to format equations. MathJax reference.


To learn more, see our tips on writing great answers.




draft saved


draft discarded














StackExchange.ready(
function ()
StackExchange.openid.initPostLogin('.new-post-login', 'https%3a%2f%2fmath.stackexchange.com%2fquestions%2f3239468%2fcomputing-the-matrix-powers-of-a-non-diagonalizable-matrix%23new-answer', 'question_page');

);

Post as a guest















Required, but never shown





















































Required, but never shown














Required, but never shown












Required, but never shown







Required, but never shown

































Required, but never shown














Required, but never shown












Required, but never shown







Required, but never shown







Popular posts from this blog

Invision Community Contents History See also References External links Navigation menuProprietaryinvisioncommunity.comIPS Community ForumsIPS Community Forumsthis blog entry"License Changes, IP.Board 3.4, and the Future""Interview -- Matt Mecham of Ibforums""CEO Invision Power Board, Matt Mecham Is a Liar, Thief!"IPB License Explanation 1.3, 1.3.1, 2.0, and 2.1ArchivedSecurity Fixes, Updates And Enhancements For IPB 1.3.1Archived"New Demo Accounts - Invision Power Services"the original"New Default Skin"the original"Invision Power Board 3.0.0 and Applications Released"the original"Archived copy"the original"Perpetual licenses being done away with""Release Notes - Invision Power Services""Introducing: IPS Community Suite 4!"Invision Community Release Notes

Canceling a color specificationRandomly assigning color to Graphics3D objects?Default color for Filling in Mathematica 9Coloring specific elements of sets with a prime modified order in an array plotHow to pick a color differing significantly from the colors already in a given color list?Detection of the text colorColor numbers based on their valueCan color schemes for use with ColorData include opacity specification?My dynamic color schemes

Tom Holland Mục lục Đầu đời và giáo dục | Sự nghiệp | Cuộc sống cá nhân | Phim tham gia | Giải thưởng và đề cử | Chú thích | Liên kết ngoài | Trình đơn chuyển hướngProfile“Person Details for Thomas Stanley Holland, "England and Wales Birth Registration Index, 1837-2008" — FamilySearch.org”"Meet Tom Holland... the 16-year-old star of The Impossible""Schoolboy actor Tom Holland finds himself in Oscar contention for role in tsunami drama"“Naomi Watts on the Prince William and Harry's reaction to her film about the late Princess Diana”lưu trữ"Holland and Pflueger Are West End's Two New 'Billy Elliots'""I'm so envious of my son, the movie star! British writer Dominic Holland's spent 20 years trying to crack Hollywood - but he's been beaten to it by a very unlikely rival"“Richard and Margaret Povey of Jersey, Channel Islands, UK: Information about Thomas Stanley Holland”"Tom Holland to play Billy Elliot""New Billy Elliot leaving the garage"Billy Elliot the Musical - Tom Holland - Billy"A Tale of four Billys: Tom Holland""The Feel Good Factor""Thames Christian College schoolboys join Myleene Klass for The Feelgood Factor""Government launches £600,000 arts bursaries pilot""BILLY's Chapman, Holland, Gardner & Jackson-Keen Visit Prime Minister""Elton John 'blown away' by Billy Elliot fifth birthday" (video with John's interview and fragments of Holland's performance)"First News interviews Arrietty's Tom Holland"“33rd Critics' Circle Film Awards winners”“National Board of Review Current Awards”Bản gốc"Ron Howard Whaling Tale 'In The Heart Of The Sea' Casts Tom Holland"“'Spider-Man' Finds Tom Holland to Star as New Web-Slinger”lưu trữ“Captain America: Civil War (2016)”“Film Review: ‘Captain America: Civil War’”lưu trữ“‘Captain America: Civil War’ review: Choose your own avenger”lưu trữ“The Lost City of Z reviews”“Sony Pictures and Marvel Studios Find Their 'Spider-Man' Star and Director”“‘Mary Magdalene’, ‘Current War’ & ‘Wind River’ Get 2017 Release Dates From Weinstein”“Lionsgate Unleashing Daisy Ridley & Tom Holland Starrer ‘Chaos Walking’ In Cannes”“PTA's 'Master' Leads Chicago Film Critics Nominations, UPDATED: Houston and Indiana Critics Nominations”“Nominaciones Goya 2013 Telecinco Cinema – ENG”“Jameson Empire Film Awards: Martin Freeman wins best actor for performance in The Hobbit”“34th Annual Young Artist Awards”Bản gốc“Teen Choice Awards 2016—Captain America: Civil War Leads Second Wave of Nominations”“BAFTA Film Award Nominations: ‘La La Land’ Leads Race”“Saturn Awards Nominations 2017: 'Rogue One,' 'Walking Dead' Lead”Tom HollandTom HollandTom HollandTom Hollandmedia.gettyimages.comWorldCat Identities300279794no20130442900000 0004 0355 42791085670554170004732cb16706349t(data)XX5557367